Vous êtes sur la page 1sur 18

5/25/13 Exam Reports

testfunda.com/LMS/Student/NewReports.aspx 1/18
1.
3 Marks
P is a prime number greater than 1000. What can be the value of remainder when the square of P is divided by 12?
1) 1 only
2) 3 only
3) 7 only
4) 1 or 3
5) 3 or 7
Solution:
Since P is a natural number, it must leave a remainder of 0, 1, 2, 3, 4 or 5 on division by 6.
It must be of the form 6x, 6x + 1, 6x + 2, 6x + 3, 6x + 4 or 6x + 5.
Out of these, 6x, 6x + 2 and 6x + 4 are all even and 6x + 3 is divisible by 3.
Since P must be prime, it cannot have any of these forms. Thus, P must be of the form (6x + 1) or (6x + 5).
Thus, the square of P has the form (36x
2
+ 12x + 1) = 12(3x
2
+ x) + 1 or (36x
2
+ 60x + 25) = 12(3x
2
+ 5x + 2) + 1
Both these leave a remainder of 1 on division by 12.
Hence, option 1.
2.
3 Marks
How many 10-digit positive integers with distinct digits are multiples of 11111?
1) 1234
2) 2345
3) 3456
4) 4567
5) None of these
Solution:
Let N = abcdefghij be a multiple of 11111 such that a, b, c, d, e, f, g, h, i and j are distinct natural numbers.
a, b, c, d, e, f, g, h, i and j will be equal to 0, 1, 2, , 9 in some order.
a + b + c + d + e + f + g + h + i + j = 0 + 1 + 2 + 3 ++ 9 = 45.
The sum of digits of N is divisible by 9.
N is divisible by 9.
Now, 11111 = 41 271
gcd( 11111, 9) = 1
N has to be divisible by 11111 and 9 both.
N is divisible by 99999.
N = abcde 100000 + fghij is divisible by 99999.
abcde + fghij is divisible by 99999.
abcde + fghij = 99999k, for some natural number k.
But abcde < 99999 and fghij < 99999
abcde + fghij < 199998
< 2 99999
Collapse All
Section I
Reports for
Numbers - Extra
Practice 5
Overview Solution Key Unattempted Questions Report

5/25/13 Exam Reports
testfunda.com/LMS/Student/NewReports.aspx 2/18
< 2 99999
abcde + fghij = 99999
a + f = b + g = c + h = d + i = e + j = 9
(a, f), (b, g), (c, h), (d, i), (e, j) can take values (0, 9), (1, 8), (2, 7), (3, 6) and (4, 5). Also, each value can swap positions.
As a = 0 is not possible, therefore, we have two cases.
Case 1: (a, f) (0, 9) or (9, 0)
(a, f) can take four possible pairs of values (1, 8), (2, 7), (3, 6) and (4, 5). Also, each value can swap positions, i.e. (a, f) can be both (1, 8)
and (8, 1).
Number of possible values of (a, f) = 4 2!
(b, g) can take four possible pairs of values if the value taken by the pair (a, f) are not considered. Also, each value can swap positions in
two ways.
Number of possible values of (b, g) = 4 2!
Similarly, number of possible values of (c, h) = 3 2! , number of possible values of (d, i) = 2 2! and number of possible values of (e, j) = 1
2!
Total possible values from case 1 = (4 4 3 2 1) (2!)
5
= 4 4! 2
5
Case 2: (a, f) = (9, 0)
(a, f) can take one value (9, 0). The values cant swap positions as a cannot be zero.
(b, g) can take four possible pairs of values (1, 8), (2, 7), (3, 6) and (4, 5). Also, each value can swap positions, i.e. (b, g) can be both (1, 8)
and (8, 1) in two ways.
Number of possible values of (b, g) = 4 2!
Similarly, number of possible values of (c, h) = 3 2! , number of possible values of (d, i) = 2 2! and number of possible values of (e, j) = 1
2!
Total possible values from case 2 = (1 4 3 2 1) (2!)
4
= 4! 2
4
Number of 10-digit positive integers with distinst natural numbers digits that are multiples of 11111
= 4 4! 2
5
+ 4! 2
4
= 3456
Hence, option 3.
3.
3 Marks
Let n be an integer such that

Compute the remainder when n is divided by 17.
1) 16
2) 7
3) 8
4) 9
5) None of these
Solution:
By Wilsons theorem, a natural number p > 1 is a prime number iff (p 1)! + 1 is divisible by p.
16! gives a remainder 1 on division by 17.
From the solution above we know that, n gives the same remainder as 31!/17 on division by 17.

5/25/13 Exam Reports
testfunda.com/LMS/Student/NewReports.aspx 3/18

= (17 + 14) (17 + 13) (17 + 12) (17 + 1) (16 15 14 2 1)
= (17 + 14) (17 + 13) (17 + 12) (17 + 1) 16!
(17 + 14) (17 + 13) (17 + 12) (17 + 1) 16! gives the same remainder as
(14! 16!) on division by 17.
By Wilson's theorem we know that , 16! Gives remainder 1 on division by 17.
14! 15 16 gives remainder 1 on division by 17.
14! 15 16 gives the same remainder as 14! 2 1 on division by 17.
14! 15 16 gives the same remainder as 14! 2 on division by 17.
14! 2 gives remainder 1 on division by 17.
14! will give remainder 8 on division by 17.
14! 16! will give remainder 8 1 = 8 on division by 17.
14! 16! will give remainder 17 8 = 9 on division by 17.
31!/17 will give remainder 9 on division by 17.
n will give remainder 9 on division by 17.
Hence, option 4.
Alternatively,
The question can be solved without using Wilson's theorem as follows.
We can rewrite the equation as


The remainder of n on division by 17 is same as

Let this remainder be R.



All the terms of this expression except the last term are divisible by 17.
R is the remainder when (14 13 12 2 1)
2
2 1 is divided by 17.
(14!)
2
2 = ((14 11)(8 2)(13 4)(12 7)(10 5)(6 3)(9 1))
2
2
= (154 16 52 84 50 18)
2
81 2
We can see that all the terms in this expression, except 81 2, are of the form 17k 1
R will be the remainder when 81 2 is divided by 17.
R = 9
Hence, option 4.
5/25/13 Exam Reports
testfunda.com/LMS/Student/NewReports.aspx 4/18
4.
3 Marks
What is the sum of the first 46 prime numbers?
1) 3266
2) 3087
3) 4226
4) 3936
5) 4227
Solution:
Let S be the sum of the first 46 prime numbers.
S = 2 + sum of 45 prime numbers greater than 2
All prime numbers greater than two are odd.
We know that the sum of even number of odd numbers is even and the sum of odd number of odd numbers is odd.
The sum of the 45 odd (prime numbers greater than 2) numbers is odd.
The sum of the 45 prime numbers greater than 2 is odd.
Sum of an odd and an even number is always an odd number.
S will be an odd number.
We can eliminate options 1, 3 and 4.
Also, we know that there are 25 prime numbers less than 100.
The sum of first 25 prime numbers = 2 + 3 + 5 + 7 + 11 + 13 + 17 + 19 + 23 + 29 + 31 + 37 + 41 + 43 + 47 + 53 + 59 + 61 + 67 + 71 +
73 + 79 + 83 + 89 + 97 = 1060
From the 26
th
to the 46
th
prime number each number is greater than 100.
The sum of numbers from the 26
th
prime number to the 46
th
prime number will be greater than 2100.
The sum of first 46 prime numbers will be greater than 3160.
We can eliminate option 2.
Hence, option 5.
5.
3 Marks
Let N = aabb be a 4 digit natural number, such that the two three-digit numbers aab and abb are both prime. What is the sum of the digits
of the smallest such N?
1) 8
2) 10
3) 12
4) 14
5) None of these
Solution:
In the prime numbers between 100 and 200, which are 101, 103, 107, 113 etc, there is only one with first two digits same, namely, 113.
But 133 is not prime.
Now, between 221 and 229 there are 3 primes 223, 227 and 229.
Also, 233 is a prime number, therefore, the smallest number satisfying the given condition is 2233.
Sum of digits of the required number = 10
Hence, option 2.
6.
3 Marks
If n is a positive prime number, and n
4
+ 3n
3
is a perfect cube, what is the sum of the lowest two possible values of n?
1) 22
2) 28
5/25/13 Exam Reports
testfunda.com/LMS/Student/NewReports.aspx 5/18
2) 28
3) 33
4) 66
5) 70
Solution:
Let n
4
+ 3n
3
= k
3
, for some integer k.
(n + 3) must also be a perfect cube, as the two other terms in the equation n
3
and k
3
are perfect cubes.
Thus, n is 3 less than a perfect cube, and is also a positive prime number.
The positive perfect cubes are, in order, 1, 8, 27, 64, 125, etc. These give corresponding values of n as 2 (negative), 5 (possible), 24 (not
prime), 61 (possible), and so on. The lowest two possible values of n are therefore 5 and 61. Their sum is 66.

Hence, option 4.
7.
3 Marks
If X and Y are natural numbers with no common prime factor and Z is the greatest common divisor of (X + Y) and (X
2
+ Y
2
) then how many
values can Z take?
1) 0
2) 1
3) 2
4) 3
5) Cannot be determined
Solution:
X and Y have no common prime factors.
This means that gcd(X, Y) = 1.
Also, X
2
+ Y
2
= (X + Y)
2
2XY

We know that Z divides (X
2
+ Y
2
) and (X + Y), and therefore, it will divide either 2 or XY.
Case 1: Z divides XY.
Z either divides X or divides Y. [ gcd(X, Y) = 1]
If Z divides X, since it divides (X + Y), it must divide Y as well and vice versa.
Z divides X and Y both.
As gcd(X, Y) = 1, therefore, Z = 1
Case 2: Z divides 2.
From this case , Z = 1 or 2.
From case 1 and case 2, Z can take two values 1 and 2.
Hence, option 3.
8.
3 Marks
Let a, b be positive integers such that a
2
2009b + 2b
2
= 0. Find a + b.
1) 1372
2) 588
3) 688
4) 784
5) 884
Solution:
a
2
+ 2b
2
= 2009b = 41 49b
As RHS is divisible by 49, so should be the LHS.
Since the LHS is divisible by 49, it leaves a remainder 0 when divided by 7.
5/25/13 Exam Reports
testfunda.com/LMS/Student/NewReports.aspx 6/18
Since the LHS is divisible by 49, it leaves a remainder 0 when divided by 7.
This is possible only if both the parts of the LHS individually leave a remainder 0 when divided by 7 or the sum of their remainders is 7.
Whenever a square is divided by 7, it can only leave 0, 1, 2, or 4 as a remainder.
Hence, the sum of their remainders can never be 7.
Hence, both the remainders have to be 0.
Hence both a and b are divisible by 7.
Let a = 7x and b = 7y, then the equation becomes x
2
+ 2y
2
= 287y
We can still note that RHS is divisible by 7 and so is the LHS.
Hence x = 7m and y = 7n, we get m
2
+ 2n
2
= 41n
m
2
= n(41 2n), now as m is positive integer, we get n < 21.
We do trial and error for n = 1 to n = 20 and check for what value of n we get RHS as perfect square.
We get n = 16 as the only solution, which gives m = 12
a = 49m = 49 12 and b = 49n = 49 16
a + b = 49 (12 + 16) = 49 28 = 1372
Hence option 1.
9.
3 Marks
A four digit number is called peculiar if the sum of the number formed by the first two digits and the last two digits is equal to the number
formed by the middle two digits (Example 1978, 19 + 78 = 97). If all peculiar numbers are arranged in ascending order, find the sum of
peculiar numbers just before and after 1978.
1) 4285
2) 2876
3) 3876
4) 4175
5) None of these
Solution:
Let abcd be a peculiar number, then
10a + b + 10c + d = 10b + c
10a + d = 9(b c)
RHS is a multiple of 9, hence LHS has to be a multiple of 9.
To find the peculiar number just before 1978, it is easy to see a = 1 and d = 8 and therefore b c = 2.
To keep the number as close to 1978, b = 8 and hence c = 6. So the number just before 1978 is 1868.
To find the peculiar number just after 1978, we need to note that 1978 is the biggest peculiar number less than 2000.
Hence a = 2, which makes d = 7 ( 27 is a multiple of 9).
This gives b c = 3, which gives b = 3 and c = 0. Hence the number is 2307.
The sum of two peculiar number is 2307 + 1868 = 4175
Hence, option 4.
10.
3 Marks
What is the remainder when the series 1
2
+ 2
3
+ 3
4
+ + 9
10
is divided by 5?
1) 0
2) 1
3) 2
4) 3
Solution:
5/25/13 Exam Reports
testfunda.com/LMS/Student/NewReports.aspx 7/18
Solution:
Using the cyclicity property we calculate the last digit of each term in the above series.

The sum of all the last digits = 35, which is divisible by 5.
So the remainder when 1
2
+ 2
3
+ 3
4
+ + 9
10
is divided by 5 is 0.
Hence, option 1.
11.
3 Marks
Which of the following numbers cannot be written as a sum of squares of any four integers?
1) 310
2) 123
3) 59
4) 67
5) None of these
Solution:
Lagranges four square theorem states that, Every positive integer can be expressed as the sum of four squares of integers.
Consider the number 310, since we have to express it as the sum of squares of four integers we will first subtract the square of the number
closest to it i.e. 17
2
.
310 289 = 21
Now we will subtract the square of the number closest to 21 i.e. 4
2
21 16 = 5
Again we now subtract 2
2
and then 1
2
and we get a zero.
Hence, 310 = 17
2
+ 4
2
+ 2
2
+ 1
2
Similarly,
123 = 11
2
+ 1
2
+ 1
2
+ 0
2
59

= 7
2
+ 3
2
+ 1
2
+ 0
2
67 = 8
2
+ 1
2
+ 1
2
+ 1
2
Hence, option 5.
12.
3 Marks
If X = 2891 2892 2893 ... 2898 2899 2900, then what is the remainder when X is divided by 17?
1) 0
2) 7
3) 14
4) 4
5) 10
5/25/13 Exam Reports
testfunda.com/LMS/Student/NewReports.aspx 8/18
Solution:
2890 = 289 10 = 17 17 10
The remainder when 2891 is divided by 17 is 1
This can be represented as,
2891 1(mod 17)
Similarly, 2892 2(mod 17)
2893 3(mod 17)
and so on.
X (1 2 3 ... 9 10)(mod 17)
X (6! 56 90)(mod 17)
When 6! (= 720) is divided by 17, the remainder is 6
Similarly when 56 and 90 are divided by 17 the remainders are 5 and 5 respectively.
X (6 5 5)(mod 17)
X 150(mod 17)
X 14(mod 17)
Hence, option 3.
13.
3 Marks
What maximum power of 5 divides S!?
where S is the sum of digits of (4 + 44 + 444 + 9 terms)
1) 5
2) 15
3) 10
4) 8
5) None of these
Solution:
Let X = (4 + 44 + 444 + 9 terms)
X = 4(1 + 11 + 111 + 9 terms)
Now,
1 + 11 = 12 (2 terms)
1 + 11 + 111 = 123 (3 terms)
Similarly,
1 + 11 + 111 + 9 terms = 123456789
X = 4(123456789) = 493827156
Sum of digits of X is 45.
S = 45
We need to find out maximum power of 5 that exactly divides 45!
[45/5] + [45/5
2
] = 9 + 1 = 10
10
th
power of 5 divides 45!
Hence, option 3.
14.
3 Marks
Deepak was given a task to distribute 400 gifts amongst 11 people in such a manner that no two people get same number of gifts and no
two pairs of any two people should have same number of gifts and Deepak could take remaining gifts himself. What is the maximum number
of gifts that Deepak can get?
5/25/13 Exam Reports
testfunda.com/LMS/Student/NewReports.aspx 9/18
of gifts that Deepak can get?
1) 35
2) 25
3) 40
4) 20
5) None of these
Solution:
As Deepak has to distribute the gifts among 11 people and remaining gifts will be taken by Deepak himself, Deepak will try to maximize the
number of gifts that he will get.
Deepak will try to minimize the number of gifts that he will distribute.
For that he will give 1 gift to the first person, 2 gifts to the second and 3 gifts to the third.
Now, he cannot give 4 gifts to the 4
th
person as in that case, number of gifts with 1
st
and 4
th
person will be equal to number of gifts with 2
nd
and 3
rd
person i.e. 1 + 4 = 2 + 3
Now he can give 5 gifts to the 4
th
person, and so on.
By trial and error we find out that Deepak saves maximum gifts for himself if he distributes the gifts in following manner.
1, 2, 3, 5, 8, 13, 21, 30, 39, 53 and 74.
The total number of gifts that Deepak can take himself = 400 (1 + 2 + 3 + 5 + 8 + 13 + 21 + 30 + 39 + 53 + 74)
= 400 249 = 151
Hence, option 5.
15.
3 Marks
The country of Tialmoner has a unique six-stringed musical instrument. The frequencies of the strings decrease from the first string to the
sixth string. A visitor to this nation was given a tuner and the following instructions:
I. The frequency of the fifth string is a three-digit prime number not greater than 125.
II. The fifth and fourth strings share a common harmonic overtone of 15478 Hz.
III. Satisfying the above conditions, the fourth string has the least possible frequency.
IV. The second string has a frequency of 241 Hz.
V. The last five strings are placed at intervals such that the intervals are part of an arithmetic progression
VI. The first string has a frequency that is a harmonic overtone of that of the sixth.
The visitor successfully tuned the instrument. From the following options, what is a possible frequency of the first string?
(A harmonic overtone is an integer multiple of a certain frequency. The first overtone of 55 Hz is 110 Hz and so on...)
1) 336
2) 262
3) 174
4) 344
5) 348
Solution:
First, we consider conditions 1 and 2.
There are only five possible primes between 100 and 125: 101, 103, 107, 109 and 113. We check which of these is a prime factor of 15478.
101 is NOT a prime factor.
103 is NOT a prime factor.
107 is NOT a prime factor.
109 IS a prime factor of 15478. (109 71 2 = 15478)
The fifth string has a frequency of 109 Hz.
Consider condition 3.
To find the frequency of the fourth string, we need to find the prime factors of 15478 other than 109. We will then determine the least
common multiple of these factors which will give us a base frequency.

5/25/13 Exam Reports
testfunda.com/LMS/Student/NewReports.aspx 10/18
The factors of 15478 are 1, 2, 71, 109, 142, 218, 7739 and 15478.
Now, the frequency of the fourth string will be greater than the fifth (109).
The least possible value for the frequency of the fourth string is 142 (= 71 2).
Consider condition 4 and 5. The frequency interval between the fourth and fifth strings is (142 109 =) 33 Hz.
Also, the interval between the fourth and the second string is 99 Hz.
For the condition that the intervals form a progression, we can conclude that the interval between the third and the fourth strings is 44 Hz
while that between the second and the third is 55 Hz such that the frequency of the third string is 186 Hz.
Using this information, we can see that the difference between the fifth and sixth strings is 22 Hz.
The frequency of the sixth string is 87 Hz.
6
th
: 87 Hz
5
th
: 109 Hz
4
th
: 142 Hz
3
rd
: 186 Hz
2
nd
: 241 Hz
Since the frequency of the first string is a harmonic overtone of that of the sixth, we check the options provided to see which one is divisible
by 87.
348 is divisible by 87.
Hence, option 5.
16.
3 Marks
1000 students appeared for the talent search examination NCERT. The examination had 1000 questions. During checking of the papers, a
very peculiar trend was noticed. The first student attempted all the questions, the second student attempted questions with question
numbers which were multiples of 2, the third student attempted the questions with question numbers which were multiples of 3 and so on.
Let f(n) = the number of students who attempted the question number n
For example, f(10) represents the number of students who attempted the question number 10 and f(10) = 4 because four students (1
st
, 2
nd
,
5
th
and 10
th
) attempted the question.
For how many questions was f(n) odd?
1) 234
2) 31
3) 78
4) 101
5) None of these
Solution:
The first student attempted all the questions i.e. question number 1, 2, 3, ..., 999, 1000.
The second student attempted the questions with numbers which were multiple of 2 i.e. question number 2, 4, 6, ..., 998, 1000.
The third student attempted the questions with numbers which were multiple of 3 i.e. question number 3, 6, 9, 12, ..., 999.
And so on.
Here we can see that the number of students who have attempted a particular question is the number of factors (including the number itself
and 1) it has. For example, f(24) = 8 as the question number 24 was attempted by 8 students (1
st
, 2
nd
, 3
rd
, 4
th
, 6
th
, 8
th
, 12
th
and 24
th
student).
We have to find the question numbers for which the number of factors are odd.
We know that any number 'n' can be expressed in the form,
n = 2
(b)
3
(c)
5
(d)
... where 2, 3, 5,.... are prime numbers
The number of factors for n = (b + 1)(c + 1)(d + 1)...
To have odd number of factors each of (b + 1), (c + 1), (d + 1), ... should be odd.
5/25/13 Exam Reports
testfunda.com/LMS/Student/NewReports.aspx 11/18
b, c, d, ... all should be even.
n is a perfect square.
The questions with numbers which were perfect squares were attempted by odd number of students.
The problem reduces to finding out the number of perfect squares in the range 1 - 1000.
There are 31 perfect squares between 1 to 1000 including 1 and 1000.
Hence, option 2.
17.
3 Marks
Find the highest power of 23 in the following.
1001 2001 3001 4001 1002 2002 3002 4002 1003 1099 2099 3099 4099
1) 57
2) 20
3) 19
4) 18
5) 27
Solution:
1001 2001 3001 4001 1002 2002 3002 4002 1003 1099 2099 3099 4099
= (1001 ... 1099) (2001 ... 2099) (3001 ... 3099) (4001 ... 4099)

We will find the highest power of 23 in each case.


Similarly, highest powers of 23 in 2099!, 2000!, 3099!, 3000!, 4099! and 4000! are 94, 89, 139, 135, 185 and 180 respectively.
Highest power of 23 in 1001 2001 3001 4001 1002 2002 3002 4002 1003 1099 2099 3099 4099
= (49 44) + (94 89) + (139 135) + (185 180)
= 19
Hence, option 3.
18.
3 Marks
How many 2 digit numbers are there such that absolute difference of the square of the 2 digits is a prime number?
1) 21
2) 12
3) 10
4) 14
5) 16
Solution:
Let the two digit number be represented as ab.
a
2
b
2
= (a b)(a + b)
Now, |a
2
b
2
| will be a prime number only when |a b| = 1 and (a + b) comes out to be a prime number
Two digit numbers in which |a b| = 1 are 10, 12, 21, 23, 32, 34, 43, 45, 54, 56, 65, 67, 76, 78, 87, 89, 98
From these numbers, the numbers in the form of 'ab' in which (a + b) comes out to be a prime number are 12, 21, 23, 32, 34, 43, 56, 65,
67, 76, 89 and 98.
5/25/13 Exam Reports
testfunda.com/LMS/Student/NewReports.aspx 12/18
There are 12 two digit numbers such that absolute difference of the square of the two digits results in a prime number.
Hence, option 2.
19.
3 Marks
1) 0
2) 1
3) 2
4) (n 2)
5) (n 1)
Solution:


=
2n
C
n 1
This has to be an integer, since it is a binomial coefficient, or the number of ways of selecting (n 1) objects out of 2n objects.
Now, n and (n + 1) cannot have any factor in common for n > 1.
Since the binomial coefficient is an integer,


Note: This is true for all n > 1, and not merely n > 53. The answer is 0.
Hence, option 1.
20.
3 Marks
If n = 1 + x, where x is the product of 4 consecutive natural numbers, then n is:
I. an odd number.
II. an even number.
III. a prime number.
IV. a perfect square.
1) II only
2) III and IV only
3) I and IV only
4) I only
5) None of these
Solution:
In 4 consecutive natural numbers, there will be at least two even numbers.
x is even and x + 1 is odd.
Statement I is true and statement II is false.
To decide whether n is prime or not, just try the first four natural numbers, which give a product of 24, and therefore, n = 25
Therefore, n is not prime, but it could be a perfect square.
5/25/13 Exam Reports
testfunda.com/LMS/Student/NewReports.aspx 13/18
To prove convincingly that n is a perfect square, try the product of any 4 consecutive natural numbers, say (y 1), y, (y + 1) and (y + 2).
n = (y 1) y (y + 1) (y + 2)


n is a perfect square.
Hence, option 3.
21.
3 Marks

Where, n is the smallest odd prime number, p and q are integers and p < q, then how many integer values can x take?
1) 0
2) 1
3) 2
4) 3
5) 4
Solution:
n is the smallest odd prime number that is 3.


If x has to take integer values, the only way it is possible is if x
2
= 9.
x = 3 or 3
x can take 2 integer values.
Hence, option 3.
22.
3 Marks
Let x and y be positive integers such that (x y) and (x + y) are prime. Then which of the following statements is/are true?
I. x
2
+ y
2
cannot be prime.
II. x
2
+ y
2
cannot be even.
III. x
2
y
2
may be even or odd.
1) I only
2) II only
3) III only
4) I and III only
5) I and II only
Solution:
If x and y are integers, then (x y) and (x + y) are either both even, or both odd. Since (x y) and (x + y) are prime numbers, therefore, both
are odd, and so one of x and y needs to be an even number and the other has to be an odd number.

(x
2
+ y
2
) and (x
2
y
2
) will always be odd.
II is true and III is false.
Now consider x = 5 and y = 2
x
2
+ y
2
= 25 + 4 = 29 which is a prime number.
I is false.
Hence, option 2.
5/25/13 Exam Reports
testfunda.com/LMS/Student/NewReports.aspx 14/18
Hence, option 2.
23.
3 Marks
X is the set of all numbers n such that 10 < n 50 and the remainder when (n 1)! is divided by n is not zero. The number of numbers in
the Set X is equal to:
1) 10
2) 16
3) 11
4) 17
5) 5
Solution:
The remainder when (n 1)! is divided by n is not zero only if n is a prime number.
The prime numbers greater than 10 and less than 50 are (11, 13, 17, 19, 23, 29, 31, 37, 41, 43, 47).
The total number of numbers in the set is 11.
Hence option 3.
24.
3 Marks
X is a number of more than 4 digits. The digits of X increase from left to right (thus, each digit is greater than the digit immediately to the left
of it). What can be said about the sum of digits of the number 9X?
1) Can only be 9
2) Can be 3 or 9
3) Can be 3, 6 or 9
4) Can be 9 or 18
5) Can be 0 or 9
Solution:
Let X be of the form (from left to right) X
1
X
2
X
3
X
n
, where each X
i
is a digit.
Then,10X is of the form X
1
X
2
X
3
X
n
0
9X = 10X X
Now, the rightmost digit of 9X will be equal to (10 X
n
) because there will be a borrow from the digit immediately to its left. Hence the digit
to the left of the rightmost digit of 9X will be (X
n
X
n 1
1).
There will be no more borrows from any other digit of 10X since the digits of X increase strictly from left to right.
9X will be of the form X
1
(X
2
X
1
) (X
3
X
2
) ... (X
n
X
n 1
1)(10 X
n
)
If we take the sum of these digits, there is pairwise cancellation in each term.
At the end, all that is left is 10 1 = 9
The sum of digits of 9X will always be 9.
Hence, option 1.
25.
3 Marks
What is the units digit of 1
1
+ 2
2
+ 3
3
+ + 2008
2008
?
1) 8
2) 6
3) 4
4) 2
5) 0
Solution:
We consider the numbers 10 at a time, i.e. 1
1
+ 2
2
+ ... + 10
10
, and so on.
Now, if we know the cyclicity of all the numbers from 0 to 9, we can conclude the following:
5/25/13 Exam Reports
testfunda.com/LMS/Student/NewReports.aspx 15/18
Now, if we know the cyclicity of all the numbers from 0 to 9, we can conclude the following:
All numbers ending in 1, 5, 6 or 0 will end with the same digits irrespective of the power they are raised to.
All numbers ending in 4 are raised to an even power here, so their units digit will be 6.
All numbers ending in 9 are raised to odd powers, so their units digit will be 9.
For numbers ending in 2 or 8, the power is alternately of the form 4n or (4n + 2) (for example: 2, 22, 42, ... and 18, 38, 58, ... are of the form
(4n + 2), while 12, 32, 52, ... and 8, 28, 48, ... are of the form 4n).
So, for numbers ending in 2, the units digit is alternately 4 or 6, while for the numbers ending in 8, the units digit is alternately 6 or 4.
Similarly, for numbers ending in 3 or 7, the power is alternately of the form (4n + 3) or (4n + 1), and so the units digit is 7 and 3 respectively
for numbers ending in 3, or 3 and 7 respectively for numbers ending in 7.
For every 10 numbers, the sum of the units digits is 1 + 4 + 7 + 6 + 5 + 6 + 3 + 6 + 9 + 0, or 1 + 6 + 3 + 6 + 5 + 6 + 7 + 4 + 9 + 0.
In both cases, the sum of these 10 digits is 47.
The sum from 1 to 2008 will be 47 200 + (1 + 4 + 7 + 6 + 5 + 6 + 3 + 6) = 9438
Since we are only concerned with the units digit, the answer is 8.
Hence, option 1.
26.
3 Marks
In a country called Septophobasia, the number 7 was once banned from use. Due to this, the counting system (which was the decimal
system) had to be revamped. To this end, officials began writing all the allowed numbers in order, obviously skipping all those which featured
the digit '7' anywhere in them. What was the 59056
th
number to be written?
1) 100008
2) 334114
3) 200000
4) 87436
5) 99056
Solution:
A quick way to do this problem is to consider the first 10 numbers. Only 9 of them will be written in Septophobasia (since 7 will be left out).
The 9
th
number to be written will be 10.
Now, consider the first 100 numbers. 10 numbers with 7 in the units place will be left out, as will 10 numbers with 7 in the tens place. But
one number has 7 in both the units and tens places (77).
19 numbers will be left out, and the 81
st
number will be 100.
Looking at the trends, we can see that the (9
n
)
th
number to be written will be (10
n
). 59049 (9
5
) is the closest power of 9 to 59056, so the
59049
th
number to be written will be 100,000.
Counting forward from there, the 59056
th
number to be written is 100,008.
Hence, option 1.
Alternatively,
Since the Septophobasians have only 9 digits to choose from after banning 7, therefore their 9th number will be 10, (9
2
)
th
number will be
10
2
, (9
n
)
th
number will be 10
n
, and so on.
Hence, option 1.
27.
3 Marks
Let C(n) be defined as the smallest positive integer that can be written as the sum of two cubes (of positive or negative integers) in n
different ways. For example, C(2) = A
3
+ B
3
= X
3
+ Y
3
, where A, B, X, Y are integers. 30 < C(2) < 100, |ABXY| =
1) 360
2) 180
3) 120
4) 240
5) 420
5/25/13 Exam Reports
testfunda.com/LMS/Student/NewReports.aspx 16/18
Solution:
Possible values of A and B, keeping in mind that 30 < C(2) < 100, are:
[A = 1, B = 4; A
3
+ B
3
= 65], [A = 2, B = 3; A
3
+ B
3
= 35], [A = 2, B = 4; A
3
+ B
3
= 72],
[A = 3, B = 3; A
3
+ B
3
= 54], [A = 3, B = 4; A
3
+ B
3
= 91], [A = 4, B = 1; A
3
+ B
3
= 63],
[A = 4, B = 2; A
3
+ B
3
= 56], [A = 4, B = 3; A
3
+ B
3
= 37], [A = 5, B = 3; A
3
+ B
3
= 98],
[A = 5, B = 4; A
3
+ B
3
= 61], [A = 6, B = 5; A
3
+ B
3
= 91]
We see that the only sum which occurs twice is 91, and hence that C(2) must be 91, since it is the smallest number in this list expressible
as the sum of two cubes in two different ways.
91 = 3
3
+ 4
3
= 6
3
5
3
|ABXY| = 3 4 5 6 = 360
Hence, option 1.
28.
3 Marks
An eight digit number is the largest possible number such that the number formed by the four leftmost digits is twice the number formed by
the four rightmost digits. Which of the following is not a prime factor of this eight digit number?
1) 3
2) 59
3) 113
4) 4999
5) 6667
Solution:
The eight digit number satisfying the given condition is 99984999 (9998 is twice as large as 4999).
Now, the biggest hint for finding factors of this number is in the question itself.
9998 = 4999 2
Multiplying both the sides by 10000, we get,
99980000 = 4999 20000
Adding 4999 to both the sides, we get,
99984999 = 4999 20000 + 4999
99984999 = 4999 20001
It is easy to see that 20001 is divisible by 3, so we divide it by 3 to find that 20001 = 6667 3
Now, instead of trying to determine rigorously whether 4999 and 6667 are prime or not, we take a hint from option 2 and option 3, and see
that 59 113 = 6667
6667 is not prime.
Hence, option 5.
29.
3 Marks
For any two natural numbers x and y, if x
2
+ y
2
is divisible by 21, what are the remainders given by x and y respectively on division by 21?
1) 3, 4
2) 2, 5
3) 0, 0
4) 1, 6
5) None of these
Solution:
Note that if x gives a remainder of r on division by any number, then x
2
will give a remainder of r
2
. In this case, if x
2
+ y
2
is divisible by 21,
that means it is divisible by 3 as well as by 7.
Now, all natural numbers, on division by 3, give a remainder of 0, 1 or 2. Therefore, their squares, give a remainder of 0 or 1 (since a
5/25/13 Exam Reports
testfunda.com/LMS/Student/NewReports.aspx 17/18
Now, all natural numbers, on division by 3, give a remainder of 0, 1 or 2. Therefore, their squares, give a remainder of 0 or 1 (since a
remainder of 4 is further divisible by 3, giving a remainder of 1).
Therefore, the sum of two squares will give a remainder of 0 (0 + 0), 1 (0 + 1 or 1 + 0) or 2 (1 + 1). In other words, when the sum of two
squares gives a remainder of 0 on division by 3, the only way this can happen is if both the squares themselves are divisible by 3, and
therefore, both the original numbers are divisible by 3.
Similarly, all natural numbers, on division by 7, give a remainder of 0, 1, 2, 3, 4, 5 or 6. Therefore, their squares give a remainder of 0, 1, 2 or
4 (prove this). Now, note that the only way the sum of two squares can give a remainder of 0 on division by 7 is if both squares are
themselves divisible by 7, and so, both the original numbers are divisible by 7.
Thus, if x
2
+ y
2
is divisible by 21, both x and y will be divisible by 21.
Hence, option 3.
Note: Elimination of options may be tried, but it will not help in choosing between options 3 and 5.
30.
3 Marks
Akash runs a business of exporting mangoes. He has 8649 Alphonso mangoes and 7688 Kesar mangoes in his warehouse. He wants to
pack these into boxes of uniform size in a single-layer square arrangement such that the number of mangoes in each row in a box should
be equal to the number of mangoes in each column, and the total number of mangoes in a box should be equal for all boxes. Also, he wants
to ensure that each box contains only one type of mango, and that no mangoes are left out at the end of this packing. How many mangoes
will have to be packed into each box?
1) 529
2) 729
3) 841
4) 961
5) None of these
Solution:
The given conditions tell us that the number of mangoes in each box will have to be a common factor of 7688 and 8649, and since the
mangoes have to be arranged in a square in each box, this number should be a perfect square.
Instead of trying to factorize 7688 and 8649 directly, we can simplify our task if we notice that a common factor of 7688 and 8649 will also
have to be a factor of their difference,
i.e. (8649 7688) = 961
The only factors that 961 has are 1, 31 and 961, and we can verify that 7688 = 961 8 and 8649 = 961 9
The required number is 961.
Hence, option 4.
31.
3 Marks
Dr. Numerez is reading the newspaper in his living room when the doorbell rings. He opens it to find a salesman trying to sell him a new
product. Dr. Numerez is not interested, but when the salesman insists, he tells him, "You can go to my brother. He might be interested in
buying this. His house is in this very block, and its number is composite, but relatively prime to the number of my house. You can ask me
just one more question, which I should be able to answer with a 'yes' or 'no', and then leave." The salesman knows that Dr. Numerez's
house is number 10 in the block, which has houses numbered 1 to 26.
Assuming that Dr. Numerez will answer him correctly, which of the following questions should the salesman ask to uniquely determine the
number of his brother's house?
1) Is the number a perfect square?
2) Does the number have exactly 4 distinct divisors?
3) Does the number have exactly 3 distinct divisors?
4) Is the number lesser than the number of your house?
5) Any of these
Solution:
Dr. Numerez has already told him that the number is composite, but relatively prime to his house number.
Looking for such numbers from 1 to 26, we find only two: 9 and 21.
(These two numbers can be found out faster by using the knowledge that none of the even or prime numbers would satisfy the conditions
stated by Dr. Numerez.)
9 is a perfect square while 21 is not.
5/25/13 Exam Reports
testfunda.com/LMS/Student/NewReports.aspx 18/18
9 is a perfect square while 21 is not.
9 has 3 distinct divisors while 21 has 4.
9 is lesser than 10 while 21 is more.
Any of the given questions will help the salesman uniquely determine the answer.
Hence, option 5.
32.
3 Marks
If a
2
b
2
= 2010, then what is the number of possible solutions (a, b) such that a and b are natural numbers?
1) 0
2) 1
3) 2
4) 12
5) 14
Solution:
It is easier to see through this problem if a
2
b
2
is written as (a + b)(a b).
Now, a and b can be either even or odd. If both are even or both are odd, then (a + b) and (a b) are both even, in which case their
product would be divisible by 4.
If one of them is even and the other is odd, then (a + b) and (a b) are both odd, in which case their product would be odd.
2010 is neither odd, nor divisible by 4.
The answer is 0.
Hence, option 1.

Vous aimerez peut-être aussi